Last visit was: 24 Apr 2024, 19:18 It is currently 24 Apr 2024, 19:18

Close
GMAT Club Daily Prep
Thank you for using the timer - this advanced tool can estimate your performance and suggest more practice questions. We have subscribed you to Daily Prep Questions via email.

Customized
for You

we will pick new questions that match your level based on your Timer History

Track
Your Progress

every week, we’ll send you an estimated GMAT score based on your performance

Practice
Pays

we will pick new questions that match your level based on your Timer History
Not interested in getting valuable practice questions and articles delivered to your email? No problem, unsubscribe here.
Close
Request Expert Reply
Confirm Cancel
SORT BY:
Date
User avatar
Senior Manager
Senior Manager
Joined: 25 Jun 2011
Status:Finally Done. Admitted in Kellogg for 2015 intake
Posts: 396
Own Kudos [?]: 16650 [21]
Given Kudos: 217
Location: United Kingdom
Concentration: International Business, Strategy
GMAT 1: 730 Q49 V45
GPA: 2.9
WE:Information Technology (Consulting)
Send PM
Most Helpful Reply
Math Expert
Joined: 02 Sep 2009
Posts: 92900
Own Kudos [?]: 618824 [6]
Given Kudos: 81588
Send PM
General Discussion
Manager
Manager
Joined: 22 Feb 2012
Posts: 74
Own Kudos [?]: 67 [3]
Given Kudos: 25
Schools: HBS '16
GMAT 1: 670 Q42 V40
GMAT 2: 740 Q49 V42
GPA: 3.47
WE:Corporate Finance (Aerospace and Defense)
Send PM
Manager
Manager
Joined: 06 Jan 2012
Status:May The Force Be With Me (D-DAY 15 May 2012)
Posts: 165
Own Kudos [?]: 2093 [4]
Given Kudos: 33
Location: India
Concentration: General Management, Entrepreneurship
Send PM
Re: After winning 50 percent of the …first 30 matches she played [#permalink]
4
Kudos
Hi ,

I solved it a little differently so might help

First 30 Games Remaining Games Total Games
Won 15 Games x Games 15 + x Games
Lost 15 Games 0 Games 15 Games
Total : 30 Games x Games 30 + x Games

Statement 1 : 15+x = .75 ( x+ 30 )
Hence sufficient

Statement 2 : 0.50 ( x+30 ) = 15 + 15

hence sufficient

I hope this helps :-)
avatar
Intern
Intern
Joined: 14 Feb 2014
Posts: 9
Own Kudos [?]: 16 [0]
Given Kudos: 11
GMAT Date: 06-21-2014
Send PM
Re: After winning 50 percent of the …first 30 matches she played [#permalink]
After winning 50 percent of the …first 30 matches she played, Hortense won all of her remaining matches. How many total matches did she win?

(1) Hortense won 75 percent of the matches she played. Say # of remaining matches is R: 30*0.5+R*1=(30+R)*0.75, we have only one unknown R, hence we can solve for it. Sufficient.

(2) If Hortense had won 50 percent of the total number of matches she played, she would have lost 15 more total matches --> Hortense won 50% of his first 30 matches and 100% of the remaining matches, in order to win 50% of total matches she should have won 50% of the remaining matches (instead of 100%, so 50% less). So 50% losses in the remaining matches (R) result in 15 more losses: 0.5*R=15 --> R=30 --> total matches won: 30*0.5+30*1=45. Sufficient.

Hi There - Is there any way you can explain the second point again please
Math Expert
Joined: 02 Sep 2009
Posts: 92900
Own Kudos [?]: 618824 [0]
Given Kudos: 81588
Send PM
Re: After winning 50 percent of the …first 30 matches she played [#permalink]
Expert Reply
Raghunathnallu wrote:
After winning 50 percent of the …first 30 matches she played, Hortense won all of her remaining matches. How many total matches did she win?

(1) Hortense won 75 percent of the matches she played. Say # of remaining matches is R: 30*0.5+R*1=(30+R)*0.75, we have only one unknown R, hence we can solve for it. Sufficient.

(2) If Hortense had won 50 percent of the total number of matches she played, she would have lost 15 more total matches --> Hortense won 50% of his first 30 matches and 100% of the remaining matches, in order to win 50% of total matches she should have won 50% of the remaining matches (instead of 100%, so 50% less). So 50% losses in the remaining matches (R) result in 15 more losses: 0.5*R=15 --> R=30 --> total matches won: 30*0.5+30*1=45. Sufficient.

Hi There - Is there any way you can explain the second point again please


Please elaborate what exactly was confusing there. Thank you.

Meanwhile check similar questions to practice:
after-winning-80-percent-of-the-fi-rst-40-games-it-played-129338.html
a-chess-player-won-25-percent-of-the-first-20-games-152608.html
after-winning-80-of-his-first-40-matches-igby-won-129062.html
after-winning-50-percent-of-the-first-x-games-it-played-149675.html
after-winning-30-percent-of-the-first-50-games-it-played-142267.html
after-winning-50-percent-of-the-first-20-games-it-played-te-167045.html

Hope this helps.
avatar
Intern
Intern
Joined: 06 May 2014
Posts: 5
Own Kudos [?]: 1 [1]
Given Kudos: 33
Concentration: Marketing, Technology
GMAT 1: 660 Q49 V31
Send PM
Re: After winning 50 percent of the …first 30 matches she played [#permalink]
1
Kudos
1) she won 75% of total matches means she did not win 25% of matches , which is 50% of first 30 matches(=> 15 matches), since she won all the remaining matches .
let total matches be x , 25% of x=15
x=60 , hence sufficient

2) if for 50% win she lost 15 more matches, total match not won = 30 , total matches = 2(30) = 60
hence, sufficient
Manager
Manager
Joined: 22 Mar 2014
Posts: 79
Own Kudos [?]: 36 [0]
Given Kudos: 136
Location: United States
Concentration: Finance, Operations
GMAT 1: 530 Q45 V20
GPA: 3.91
WE:Information Technology (Computer Software)
Send PM
After winning 50 percent of the …first 30 matches she played [#permalink]
When it is said that out of 30 matches 50% of matches are won, i.e 15 matches are won. How can I assume that 15 matches were lost? There could be a tie unless specifically mentioned that no tie is possible. So in that case I assume ans would be A. What do u say guys?
Manager
Manager
Joined: 07 Aug 2018
Posts: 88
Own Kudos [?]: 251 [0]
Given Kudos: 247
Location: United States (MA)
GMAT 1: 560 Q39 V28
GMAT 2: 670 Q48 V34
Send PM
Re: After winning 50 percent of the …first 30 matches she played [#permalink]
One could also think of it in terms of ratios:

S(1) \(\frac{75}{100}=\frac{Total Games Won}{Total Games Played}=\frac{15+x}{30+x}\)
\(--> x=30\), so Total Games Won: \(15+30=45\)

S(2) \(\frac{50}{100}=\frac{Total Games Won}{Total Games Played}=\frac{15+x}{30+2x}\)
\(--> x=30\), so Total Games Won: \(15+30=45\)
User avatar
Non-Human User
Joined: 09 Sep 2013
Posts: 32657
Own Kudos [?]: 821 [0]
Given Kudos: 0
Send PM
Re: After winning 50 percent of the …first 30 matches she played [#permalink]
Hello from the GMAT Club BumpBot!

Thanks to another GMAT Club member, I have just discovered this valuable topic, yet it had no discussion for over a year. I am now bumping it up - doing my job. I think you may find it valuable (esp those replies with Kudos).

Want to see all other topics I dig out? Follow me (click follow button on profile). You will receive a summary of all topics I bump in your profile area as well as via email.
GMAT Club Bot
Re: After winning 50 percent of the …first 30 matches she played [#permalink]
Moderator:
Math Expert
92900 posts

Powered by phpBB © phpBB Group | Emoji artwork provided by EmojiOne